If you don't have time or don't feel like readings lot (or what ever reason you may have) there is an abridged version at the bottom that is much easier to read.
In this note, I will prove that for all polynomials f(x)=anxn+an−1xn−1+...+a0, with an and a0=0 and x=0 is not a root, that after being divided by GCD of the coefficients (g) or multiplied by a factor of k such that the new coefficients are both integral and co-prime, the function with the smallest possible integral inverse roots has the form f′(x)=p(a0xn+a1xn−1...+an) where p is a constant and p=1 if and only if f(x) has all group wise co-prime integer coefficients.
(In the short, I'm proving that the coefficients are reversed if the roots are inverted (AKA: r1 becomes r11)
By Vieta's, we have −anan−1=r1+r2+r3⋯+rn for f(x)
Now, assume that we didn't know that f′(x) had the reverse coefficients of f(x)
Thus f′(x)=bnxn+bn−1xn−1...+b0
However, we know that the roots of f′(x) are ri1 where 0≤i≤n.
For f′(x), we have −bnbn−1=r11+r21+r31⋯+rn1
If we multiply both the top and bottom of every fraction on the R.H.S. by every root other than the one in its denominator, we will have −(ana0)(ana1)⇒−a0a1 (This is too ugly to show in "proper" math format here. So I'll show a simpler and proper version below)
∴−a0a1=−bnbn−1
It can be easily observed that bn=p(a0) where p is a constant and bn−1=p(a1).
This process can be repeated to show that aiai+1=bn−(i)bn−(i+1) where 0≤i<n (NOTE: this is the one time where i<n rather than i≤n because there is no b−1 nor an+1
We can see that the ratio's of the ai is equal to the ratio's of the bi in reverse order. Going back to our other equation a0a1=bnbn−1. Assuming that f(x) has all group wise co-prime coefficients, a0a1 could be reducible by a factor of h. However, this leads to two contradictions: 1. every other ratio would have to be reduced by h or f(x) would be changed. 2. If every other coefficient was reduced then some would become non-integers if they were all group wise co-prime (if they weren't then this would contradict a condition earlier).
Also since we're looking for an f′(x) with the smallest possible co-prime integer coefficients, bnbn−1 should be irreducible. These statements imply that bn−1=a1 and bn=a0.
Once again, we can use this logic to prove that ai=bn−i where 0≤i≤n.
Thus we have proved that for integral coefficients of f(x), f′(x)=a0xn+a1xn−1...+an
If f(x) doesn't have integer coefficients, then we can multiply by a factor of p such that the coefficients become integral, thus our final result is f′(x)=p(a0xn+a1xn−1...+an).
Proper Version for omitted part
−bnbn−1=r11+r21+r31⋯+rn1=i=1∑n⎝⎜⎜⎜⎜⎛ri(i=1∑n(ri))⎠⎟⎟⎟⎟⎞⋅(i=1∑n(ri))1
⇒i=1∑n⎝⎜⎜⎛ri(ana0)⎠⎟⎟⎞⋅(ana0)1=i=1∑n(an1⋅ria0)⋅a0an
⇒(an1⋅(a1))⋅a0an=a0a1
Pretty version for those like myself who hate looking at ugly summations:
Polynomial ax3+bx2+cx+d has roots p,q,r.
−ab=p+q+r
ac=pq+qr+rp
−ad=pqr
Polynomial hx3+gx2+jx+k has roots p1,q1,r1
-\dfrac{g}{h}=\dfrac{1}{p}+\dfrac{1}{q}+\dfrac{1}{r}=\dfrac{(pq)}{(pq)r}+\dfrac{(rp)}{(rp)q}+\dfrac{(pq)}{(pq)r}=\dfrac{pq+qr+rp}{pqr}=\Rightarrow -\dfrac{\left(\dfrac{c}{a}\right)}{\left(\dfrac{d}{a}\right)}=\boxed{\color \pink{-\dfrac{c}{d}}}
\dfrac{j}{h}=\dfrac{1}{pq}+\dfrac{1}{qr}+\dfrac{1}{rp}=\dfrac{(r)}{(r)pq}+\dfrac{p}{(p)qr}+\dfrac{(q)}{(q)rp}=\dfrac{p+q+r}{pqr}=\dfrac{\left(\dfrac{b}{a}\right)}{\left(\dfrac{d}{a}\right)}=\boxed{\color \green{\dfrac{b}{d}}}
-\dfrac{k}{h}=\dfrac{1}{pqr}=\dfrac{1}{\left(-\dfrac{d}{a}\right)}=\boxed{\color \red{-\dfrac{a}{d}}}
Assuming a monic polynomial (we can divide by h because dividing by a constant won't change the roots)
hx3+gx2+jx+k has equivalent roots to x^3+\dfrac{g}{h}x^2+\dfrac{j}{h}x+\dfrac{k}{h}=x^3+\color \pink{\dfrac{c}{d}}x^2+\color \green{\dfrac{b}{d}}x+\color \red{\dfrac{a}{d}}
Multiplying through to rationalize the denominators by d.
dx3+cx2+bx+a
#Algebra
#NumberTheory
#Roots
#Inverse
#Easymoney
Easy Math Editor
This discussion board is a place to discuss our Daily Challenges and the math and science related to those challenges. Explanations are more than just a solution — they should explain the steps and thinking strategies that you used to obtain the solution. Comments should further the discussion of math and science.
When posting on Brilliant:
*italics*
or_italics_
**bold**
or__bold__
paragraph 1
paragraph 2
[example link](https://brilliant.org)
> This is a quote
\(
...\)
or\[
...\]
to ensure proper formatting.2 \times 3
2^{34}
a_{i-1}
\frac{2}{3}
\sqrt{2}
\sum_{i=1}^3
\sin \theta
\boxed{123}
Comments
Why not tranform the equation by the substitution x=y1 Its a 1 line proof after that.
Log in to reply
That transformation is certainly a much quicker approach. Can you add more details about what you did?
For a problem along the same lines, check out Squaring the roots of a polynomial.
I'm not sure exactly what you mean, but by your substitution, since we're solving for roots, isn't x=∞
Log in to reply
Why x=∞?
Aren't we just inverting the roots?
Log in to reply
\x=y1. And since we're finding roots y=0
Because you substituteLog in to reply
I just used y to take a different variable. I did not mean the y in y=f(x)
Log in to reply
Note: You should state that x=0 is not a solution, or that equivalently a0=0. This would also justify allowing you to divide by a0.
Log in to reply
Ahh, thanks, thats a good point.
Thanks bruh
Log in to reply
Chee Bra.